Đến nội dung

Hoang Dinh Nhat nội dung

Có 395 mục bởi Hoang Dinh Nhat (Tìm giới hạn từ 30-05-2020)



Sắp theo                Sắp xếp  

#682131 VMF's Marathon Bất Đẳng Thức Olympic

Đã gửi bởi Hoang Dinh Nhat on 27-05-2017 - 15:30 trong Bất đẳng thức và cực trị

Giải giúp mình bài này với. 

Cho a,b,c >0  a2  + b+ c= 3. Tìm GTLN của A = $\dpi{100} \LARGE \frac{a}{a^{2}+2b+3}+\frac{b}{b^{2}+2c+3}+\frac{c}{c^{2}+2a+3}$

Áp dụng BĐT AM-GM ta có: $\sum \frac{a}{a^2+2b+3}\leq \frac{1}{2}\sum \frac{a}{a+b+1}$

Ta có:$\sum \frac{a}{a+b+1}\leq 1$(1)

Thật vậy: (1)$\Leftrightarrow \sum (1-\frac{a}{a+b+1})\geq 2$
$\Leftrightarrow \sum \frac{b+1}{a+b+1}\geq 2$

$\Leftrightarrow \sum \frac{(b+1)^2}{(a+b+1)(b+1)}\geq 2$

Theo $C-S$ ta có: $\sum \frac{(b+1)^2}{(a+b+1)(b+1)}\geq \frac{(a+b+c+3)^2}{a^2+b^2+c^2+2(ab+bc+ca)+6(a+b+c)+6}=\frac{2(a+b+c+3)^2}{(a+b+c+3)^2}=2$

$\Rightarrow A\leq \frac{1}{2}$

Đạt tại: $a=b=c=1$




#677164 Violympic toán tiếng anh cấp quốc gia

Đã gửi bởi Hoang Dinh Nhat on 12-04-2017 - 11:38 trong Cuộc thi VIOlympic (Cuộc thi do Bộ giáo dục và đào tạo tổ chức)

1. Given $P(x)=(x^{2}-\frac{1}{2}x-\frac{1}{2})^{2010}.$

If P(x)=$a_{2016}x^{2016}+a_{2015}x^{2015}+...+a_{1}x+a_{0}$

Tính tổng $a_{0}+a_{2}+a_{4}+...+a_{2014}$

2. Suppose that the polynomial $f(x)=x^{5}-x^{7}-4x^{3}+2x^{2}+4x+1$ has 3 solutions $x_{1};x_{2};x_{3};x_{4};x_{5} $

the other polynomial $k(x)=x^{2}-4$. Find the value of $P=k(x_{1})\times k(x_{2})\times k(x_{3})\times k(x_{4})\times k(x_{5})$

 

bài 1: Ta xét p(1) bằng bao nhiêu là tính ra tổng đó bằng 0 nhé

Bài 2: Xét f(-4);f(4) tính ra P=-15 nhé




#677167 Violympic toán tiếng anh cấp quốc gia

Đã gửi bởi Hoang Dinh Nhat on 12-04-2017 - 12:04 trong Cuộc thi VIOlympic (Cuộc thi do Bộ giáo dục và đào tạo tổ chức)

Bạn làm rõ ra được ko

 

bạn sửa lại đề đi hình như đề sai




#677172 Violympic lớp 9 vòng 17 năm 2016 - 2017

Đã gửi bởi Hoang Dinh Nhat on 12-04-2017 - 13:13 trong Chuyên đề toán THCS

Mình đoán dấu = xảy ra tại $t^2=4$ nên thay vào thôi ,,bạn có thể dùng biệt thức delta để giải ,,có thể cũng sẽ ra !!!

 

a=?




#676714 Violympic

Đã gửi bởi Hoang Dinh Nhat on 09-04-2017 - 11:13 trong Đại số

B. 70




#696684 TOPIC thảo luận, trao đổi toán thi học sinh giỏi khối 10,11 .

Đã gửi bởi Hoang Dinh Nhat on 16-11-2017 - 17:07 trong Chuyên đề toán THPT

Bài 21: Cho một đường tròn với hai dây $AB$ và $CD$ không song song. Đường vuông góc với $AB$ kẻ từ $A$ cắt đường vuông góc với $CD$ kẻ từ $C$ và từ $D$ lần lượt tại $M, P$. Đường vuông góc với $AB$ kẻ từ $B$ cắt đường vuông góc với $CD$ kẻ từ $C$ và $D$ lần lượt tại $Q$ và $N$. Chứng minh rằng các đường thẳng $AD, BC, MN$ đồng quy và các đường thẳng $AC, BD, PQ$ cũng đồng quy.




#696544 TOPIC thảo luận, trao đổi toán thi học sinh giỏi khối 10,11 .

Đã gửi bởi Hoang Dinh Nhat on 13-11-2017 - 17:45 trong Chuyên đề toán THPT

Ủng hộ Topic một bài phương trình hàm.

Bài 13: Tìm tất cả các hàm số $f:\mathbb{R}\rightarrow \mathbb{R}$ thỏa mãn: $f(xf(y)+f(x))=2f(x)+xy, \forall x, y\in \mathbb{R}.$

Bài này đã từng được đăng ở diễn đàn. Xin được trích dẫn lời giải:

Cố định $x$ trong $(1)$ dễ dàng suy ra được $f$ song ánh. Do đó tồn tại $b$ để $f(b)=0$. Đặt $f(0)=a$.

Trong $(1)$ cho $x=b,y=0$ :

$$f(ab)=0=f(b)\Rightarrow ab=b$$

Suy ra $a=1$ hoặc $b=0$.

1) Trường hợp 1 : Nếu $b=0$ tức $f(0)=0$

Thì trong $(1)$ thay $y=0$ :

$$f(f(x))=2f(x),\;\forall x\in \mathbb{R}\Rightarrow f(x)=2x,\;\forall x\in \mathbb{R}$$

Thử lại không thỏa.

2) Trường hợp 2 : Nếu $a=1$ tức $f(0)=1$.

Trong $(1)$ cho $x=y=b$ được $b^2=a=1$ nên $b\in \left \{ 1,-1 \right \}$.

  • Nếu $b=1$ tức $f(1)=0$ thì trong $(1)$ lấy $x=1$ được :

$$f(f(y))=y,\;\forall y\in \mathbb{R}$$

Trong $(1)$ cho $y=1$ :

$$f(f(x))=2f(x)+x=2f(x)+f(f(x)),\;\forall x\in \mathbb{R}\Rightarrow f(x)=0,\;\forall x\in \mathbb{R}$$

Thử lại không thỏa.

  • Nếu $b=-1$ tức $f(-1)=0$. Trong $(1)$ cho $x=-1$ :

$$f(-f(y))=-y,\;\forall y\in \mathbb{R}\;\;\;(*)$$

Trong $(1)$ cho $y=-1$ :

$$f(f(x))=2f(x)-x,\;\forall x\in \mathbb{R}\;\;\;(2)$$

Trong $(2)$ thay $x$ bởi $-f(x)$ và sử dụng $(*)$ :

$$f(f(-f(x)))=2f(-f(x))-f(x),\;\forall x\in \mathbb{R}\Leftrightarrow f(-x)=f(x)-2x,\;\forall x\in \mathbb{R}\;\;\;\;(3)$$

Trước tiên ta sẽ tìm hàm $f:\mathbb{R}^+\rightarrow \mathbb{R}^+$ và thỏa mãn $(2)$.

Với mỗi $x\in \mathbb{R}^+$ ta xây dựng dãy $(u_n)$ :

$$u_0=x,u_1=f(x),u_n=f_n(x)$$

Khi đó từ $(2)$ ta suy ra :

$$u_{n}=2u_{n-1}-u_{n-2}$$

Phương trình sai phân của dãy $(u_n)$ là $t^2-2t+1=0$ có nghiệm kép $t=1$. Suy ra $(u_n)$ có dạng :

$$u_n=\left ( \alpha +n\beta \right ).1^n=\alpha +n\beta$$

Với $u_0=x,u_1=f(x)$ thì ta được :

$$\left\{\begin{matrix} x=\alpha \\ f(x)=\alpha +\beta \end{matrix}\right.$$

Suy ra được :

$$f(x)=x+\beta ,\;\forall x\in \mathbb{R}^+$$

Thay vào phương trình hàm ban đầu được $\beta =1$. Tức là $f(x)=x+1 ,\;\forall x\in \mathbb{R}^+$

 Kết hợp với $(3)$ :

$$f(-x)=f(x)-2x=x+1-2x=-x+1,\;\forall x\in \mathbb{R}^+$$

Mặt khác cũng có $f(0)=1$ nên ta kết luận được :

$$f(x)=x+1,\;\forall x\in \mathbb{R}$$

Thử lại thỏa mãn.

 

Đáp số : Có duy nhất một hàm số thỏa mãn đề bài là :

$$f(x)=x+1,\;\forall x\in \mathbb{R}$$




#696515 TOPIC thảo luận, trao đổi toán thi học sinh giỏi khối 10,11 .

Đã gửi bởi Hoang Dinh Nhat on 12-11-2017 - 22:30 trong Chuyên đề toán THPT

Bài 14: Trên đoạn thẳng $AB$ có độ dài $20$, người ta tô màu một số phần sao cho khoảng cách giữa hai điểm bất kì được tô màu đều khác $2$. Chứng minh rằng tổng độ dài các phần được tô màu nhỏ hơn hoặc bằng $10$.




#696510 TOPIC thảo luận, trao đổi toán thi học sinh giỏi khối 10,11 .

Đã gửi bởi Hoang Dinh Nhat on 12-11-2017 - 22:11 trong Chuyên đề toán THPT

Bài 12: (Sưu tầm) Giả sử rằng $f:\mathbb{R}\rightarrow \mathbb{R}$ là hàm số thỏa mãn điều kiện: $f(x^2-5x+1)+5f(x^2+x-5)=x^2-9, \forall x\in \mathbb{R}$ (*). Hãy tìm $f(2011)$

Trong $(*)$ thay $x$ bởi $2-x$ ta được: $f(x^2+x-5)+5f(x^2-5x+1)=x^2-4x-5, \forall x\in \mathbb{R}$ $(**)$

Nhân hai vế của $(**)$ với $-5$ rồi cộng với $(*)$ ta được: 

$-24f(x^2-5x+1)=-4x^2+20x+16, \forall x\in \mathbb{R}$

$\Leftrightarrow 6f(x^2-5x+1)=x^2-5x-4,\forall x\in \mathbb{R}$

$\Leftrightarrow 6f(x^2-5x+1)=(x^2-5x+1)-5,\forall x\in \mathbb{R}$ $(***)$

Xét phương trình: $x^2-5x+1=2011\Leftrightarrow x=\frac{5\pm \sqrt{8065}}{2}$

Thay $x=\frac{5+ \sqrt{8065}}{2}$ vào $(***)$ ta được: $6f(2011)=2011-5$$\Rightarrow f(2011)=\frac{1003}{3}$




#696461 TOPIC thảo luận, trao đổi toán thi học sinh giỏi khối 10,11 .

Đã gửi bởi Hoang Dinh Nhat on 12-11-2017 - 16:33 trong Chuyên đề toán THPT

Tiếp lửa cho topic:

Bài 11: (Sưu tầm) Tìm tất cả các hàm số  $f:\mathbb{R}\rightarrow \mathbb{R}$ thỏa mãn: $f\left ( yf(x+y)+f(x) \right )=4x+2yf(x+y)$

Bài 12: (Sưu tầm) Giả sử rằng $f:\mathbb{R}\rightarrow \mathbb{R}$ là hàm số thỏa mãn điều kiện: $f(x^2-5x+1)+5f(x^2+x-5)=x^2-9, \forall x\in \mathbb{R}$. Hãy tìm $f(2011)$




#696618 TOPIC thảo luận, trao đổi toán thi học sinh giỏi khối 10,11 .

Đã gửi bởi Hoang Dinh Nhat on 14-11-2017 - 22:35 trong Chuyên đề toán THPT

Bài 17: Tìm tất cả các hàm số $f:\mathbb{N}^*\rightarrow \mathbb{N}^*$ thỏa mãn: $f(mn)+f(m+n)=f(m)f(n)+1,\forall m,n\in \mathbb{N}^*$




#682150 Topic BẤT ĐẲNG THỨC ôn thi vào lớp 10 THPT 2017 - 2018

Đã gửi bởi Hoang Dinh Nhat on 27-05-2017 - 19:27 trong Bất đẳng thức và cực trị

Bài toán 89

1,Cho các số thực không âm a,b,c. CMR:

$\sqrt{\frac{a}{b+c}}+\sqrt{\frac{b}{c+a}}+\sqrt{\frac{c}{a+b}}\geqslant 2\sqrt{1+\frac{abc}{(a+b)(b+c)(c+a)}}$

 

2,Cho các số thực dương a,b,c.CMR:

$\frac{a}{\sqrt{b+c}}+\frac{b}{\sqrt{c+a}}+\frac{c}{\sqrt{a+b}}\geqslant \sqrt{\frac{3}{2}(a+b+c)}$

 

Bài 2:

BĐT $\Leftrightarrow (\sum \frac{a}{\sqrt{b+c}})^2\geq \frac{3(a+b+c)}{2}$

Áp dụng BĐT Holder ta có:$(\sum \frac{a}{\sqrt{b+c}})^2(\sum a(b+c))\geq (a+b+c)^3\Leftrightarrow (\sum \frac{a}{\sqrt{b+c}})^2\geq \frac{(a+b+c)^3}{\sum a(b+c)}$

Cần chứng minh: $ \frac{(a+b+c)^3}{\sum a(b+c)}\geq \frac{3(a+b+c)}{2}\Leftrightarrow  \frac{(a+b+c)^2}{\sum a(b+c)}\geq \frac{3}{2}$

$\Leftrightarrow 2(a+b+c)^2\geq 3(ab+ac+bc+ab+ac+bc)\Leftrightarrow 2(a^2+b^2+c^2)+4(ab+bc+ca)\geq 6(ab+bc+ca)\Leftrightarrow 2(a^2+b^2+c^2-ab-bc-ca)\geq 0$(đúng theo AM-GM)




#681610 Topic BẤT ĐẲNG THỨC ôn thi vào lớp 10 THPT 2017 - 2018

Đã gửi bởi Hoang Dinh Nhat on 23-05-2017 - 10:52 trong Bất đẳng thức và cực trị

Bài toán 86(sưu tầm)

Cho 3 số a,b,c dương. CM:

$\frac{(b+c+2a)^{2}}{2a^{2}+(b+c)^{2}}+\frac{(a+c+2b)^{2}}{2b^{2}+(a+c)^{2}}+\frac{(a+b+2c)^{2}}{2c^{2}+(a+b)^{2}}\leqslant 8$

 

Không mất tỉnh tổng quát chuẩn hóa: $a+b+c=3$.

BĐT cần chứng minh trở thành $\sum \frac{(3+a)^2}{2a^2+(3-a)^2}\leq 8$

Ta có BĐT: $\frac{(3+a)^2}{2a^2+(3-a)^2}\leq \frac{4a+4}{3}\Leftrightarrow 3(a-1)^2(4a+3)\geq 0$ (luôn đúng)

Áp dụng BĐT trên, ta có: $\sum \frac{(3+a)^2}{2a^2+(3-a)^2}\leq \frac{4(a+b+c)+12}{3}=8$




#679168 Topic BẤT ĐẲNG THỨC ôn thi vào lớp 10 THPT 2017 - 2018

Đã gửi bởi Hoang Dinh Nhat on 01-05-2017 - 19:56 trong Bất đẳng thức và cực trị

Bài 36: ​(sưu tầm)

 

 

 

Hình gửi kèm

  • Capture.PNG



#680541 Topic BẤT ĐẲNG THỨC ôn thi vào lớp 10 THPT 2017 - 2018

Đã gửi bởi Hoang Dinh Nhat on 13-05-2017 - 18:06 trong Bất đẳng thức và cực trị

cái đề nhìn là biết sai rồi. chỗ thì $a^3$ chỗ thì $a^2$

 

đề không sai nhé bạn




#680488 Topic BẤT ĐẲNG THỨC ôn thi vào lớp 10 THPT 2017 - 2018

Đã gửi bởi Hoang Dinh Nhat on 13-05-2017 - 10:43 trong Bất đẳng thức và cực trị

​Bài toán 64:(sưu tầm) ​Cho $a,b,c>0$ thỏa mãn $a^3+b^3+c^3=a^4+b^4+c^4$. Chứng minh rằng:

$\frac{a}{a^2+b^3+c^3}+\frac{b}{b^2+c^3+a^3}+\frac{c}{c^2+a^3+b^3}\geq 1$




#679164 Topic BẤT ĐẲNG THỨC ôn thi vào lớp 10 THPT 2017 - 2018

Đã gửi bởi Hoang Dinh Nhat on 01-05-2017 - 19:10 trong Bất đẳng thức và cực trị

$\boxed{\textbf{Bài Toán 34}}$

18279960_1846092788972773_897922199_n.pn

 

BĐT Cần chứng minh $\Leftrightarrow (a^2+1)(b^2+1)(c^2+1)-(ab+bc+ca-1)^2\geq 0$

$\Leftrightarrow a^2b^2c^2-2a^2bc+a^2-2ab^2c-2abc^2+2ab+2ac+b^2+2bc+c^2\geq 0\Leftrightarrow (abc-a-b-c)^2\geq 0$

BĐT cuối đúng nên BĐT được chứng minh




#680404 Topic BẤT ĐẲNG THỨC ôn thi vào lớp 10 THPT 2017 - 2018

Đã gửi bởi Hoang Dinh Nhat on 12-05-2017 - 17:13 trong Bất đẳng thức và cực trị

Bài toán 58(sưu tầm)

Cho 3 số dương a,b,c thỏa mãn: $a^{2}+b^{2}+c^{2}=3$

Tìm GTNN của biểu thức P=$2(a+b+c)+\left ( \frac{1}{a}+ \frac{1}{b}+\frac{1}{c}\right )$

 

Ta có: $2(a+b+c)+\frac{1}{a}+\frac{1}{b}+\frac{1}{c}=\frac{2a^2+1}{a}+\frac{2b^2+1}{b}+\frac{2c^2+1}{c}$

Ta có BĐT: $\frac{2a^2+1}{a}\geq \frac{a^2+5}{2}$ $\Leftrightarrow (2-a)(a-1)^2\geq 0$

Vì $0<a,b,c< 2$ thì BĐT phụ trên luôn đúng nên áp dụng BĐT trên, ta có: $\frac{2a^2+1}{a}+\frac{2b^2+1}{b}+\frac{2c^2+1}{c}\geq \frac{a^2+b^2+c^2+15}{2}=9$

Vậy $P\geq 9$ đạt tại $a=b=c=1$




#682334 Topic BẤT ĐẲNG THỨC ôn thi vào lớp 10 THPT 2017 - 2018

Đã gửi bởi Hoang Dinh Nhat on 29-05-2017 - 20:37 trong Bất đẳng thức và cực trị

 

 

 

3,Cho các số dương a,b,c thỏa mãn a+b+c=3. Tìm GTNN của biểu thức

P= $a^{2}+b^{2}+c^{2}+\frac{ab+bc+ca}{a^{2}b+b^{2}c+c^{2}a}$

 

 

 

 

Dễ dàng chứng minh được $a^2+b^2+c^2\geq a^2b+b^2c+c^2a$ và $a^2+b^2+c^2\geq 3$

$P\geq a^2+b^2+c^2+\frac{ab+bc+ca}{a^2+b^2+c^2}$

Đặt $a^2+b^2+c^2=x(x>0)$ rồi tách gép theo điểm rơi là được




#679967 Topic BẤT ĐẲNG THỨC ôn thi vào lớp 10 THPT 2017 - 2018

Đã gửi bởi Hoang Dinh Nhat on 08-05-2017 - 16:28 trong Bất đẳng thức và cực trị

$\boxed{\textbf{Bài Toán 52}}$ $\text{[Vasile Cirtoaje]}$ Cho $3$ số thực dương $a,b,c$. Chứng minh rằng:

\[(a^2+b^2+c^2)^2 \ge 3(a^3b+b^3c+c^3a)\]

 

BĐT cần chứng minh $\Leftrightarrow \frac{(3a^2-3ab-3ac-3b^2+6bc)^2+(-3ab+6ac+3b^2-3bc-3c^2)^2+(-3a^2+6ab-3ac-3bc+3c^2)^2}{18}$$\geq 0$

BĐT cuối luôn đúng nên BĐT được chứng minh




#679651 Topic BẤT ĐẲNG THỨC ôn thi vào lớp 10 THPT 2017 - 2018

Đã gửi bởi Hoang Dinh Nhat on 06-05-2017 - 06:15 trong Bất đẳng thức và cực trị

Bài toán 46: (Albania TST 2012)

   Tìm giá trị lớn nhất của biểu thức 

                                            $\frac{1}{x^{2}-4x+9}+\frac{1}{y^{2}-4y+9}+\frac{1}{z^{2}-4z+9}$

   Trong đó $x,y,z$ là các số thực không âm thỏa mãn $x+y+z=1$.

 

Ta có bổ đề: $\frac{1}{x^2-4x+9}\leq \frac{x}{18}+\frac{1}{9}$

$\Leftrightarrow x^3-2x^2+x=x(x-1)^2\geq 0$

BĐT cuối luôn đúng nên bổ đề được chứng minh

Áp dụng, ta có: $\sum \frac{1}{x^2-4x+9}\leq \frac{x+y+z}{18}+\frac{3}{9}=\frac{7}{18}$

Đạt tại: $x=y=0;z=1$ và các hoán vị




#681024 Topic BẤT ĐẲNG THỨC ôn thi vào lớp 10 THPT 2017 - 2018

Đã gửi bởi Hoang Dinh Nhat on 17-05-2017 - 18:38 trong Bất đẳng thức và cực trị

Bài toán 73(sưu tầm)

Cho (x+y)(z+t)+xy=88.Tìm min của P= $x^{2}+9y^{2}+6z^{2}+24t^{2}$

 

Coi lại giả thiết.

Vì ta có:

$(x+y)(z+t)+xy-88=0\Leftrightarrow 4(x+y)(z+t)+4xy-352=0$

Ta có BĐT: $x^2+9y^2+6z^2+24t^2\geq -4(x+y)(z+t)-4xy\Leftrightarrow (x+2(z+y+t))^2+(2y-z)^2+(z-4t^2)+(y-2t)^2\geq 0$ (đúng)

$\Rightarrow x^2+9y^2+6z^2+24t^2\geq -4(x+y)(z+t)-4xy=-352$ (Vô lí vì $P\geq 0$)

Nên giả thiết đúng phải là: $(x+y)(z+t)+xy=-88$ thì $P\geq 352$

Đạt tại $(x;y;z;t)=(14;-2;-4;-1)$ hoặc $(-14;2;4;1)$




#681587 Topic BẤT ĐẲNG THỨC ôn thi vào lớp 10 THPT 2017 - 2018

Đã gửi bởi Hoang Dinh Nhat on 23-05-2017 - 06:49 trong Bất đẳng thức và cực trị

Bài toán 86(sưu tầm)

Cho 3 số a,b,c dương. CM:

$\frac{(b+c+2a)^{2}}{2a^{2}+(b+c)^{2}}+\frac{(a+c+2b)^{2}}{2b^{2}+(a+c)^{2}}+\frac{(a+b+2c)^{2}}{2c^{2}+(a+b)^{2}}\geqslant 8$

Xem lại đề! Mới test thì đề sai ở bộ $(a;b;c)=(18;1;9)$.




#680559 Topic BẤT ĐẲNG THỨC ôn thi vào lớp 10 THPT 2017 - 2018

Đã gửi bởi Hoang Dinh Nhat on 13-05-2017 - 19:57 trong Bất đẳng thức và cực trị

Bài 65:chứng minh với mọi số thực ta có BDT: $\frac{x^2}{(y-z)^2}+\frac{y^2}{(x-z)^2}+\frac{z^2}{(x-y)^2}\geq 2$

 

BĐT cần chứng minh $\Leftrightarrow \frac{(x^3-x^2y-x^2z-xy^2+3xyz-xz^2+y^3-y^2z-yz^2+z^3)^2}{(y-x)^2)(z-x)^2(z-y)^2}$$\geq 0$

BĐT cuối luôn đúng nên BĐT được chứng minh




#680609 Topic BẤT ĐẲNG THỨC ôn thi vào lớp 10 THPT 2017 - 2018

Đã gửi bởi Hoang Dinh Nhat on 14-05-2017 - 08:05 trong Bất đẳng thức và cực trị

Bài toán 66(sưu tầm)

Cho các số dương x,y,z thỏa mãn $x^{2}+y^{2}+z^{2}\geqslant 3$. CMR:

$\frac{x^{5}-x^{2}}{x^{5}+y^{2}+z^{2}}+\frac{y^{5}-y^{2}}{y^{5}+z^{2}+x^{2}}+\frac{z^{5}-z^{2}}{z^{5}+x^{2}+y^{2}}\geqslant 0$

 

Bất đẳng thức cần chứng minh tương đương với $\sum \frac{1}{x^5+y^2+z^2}\leq \frac{3}{x^2+y^2+z^2}$

Từ đây ta chỉ cần xét trương hợp: $x^2+y^2+z^2=3$ nên bất đẳng thức cần chứng minh trở thành:

$\sum \frac{1}{x^5-x^2+3}\leq 1$

Theo AM-GM, ta có: $x^5=\frac{x^6}{x}\geq \frac{2x^6}{x^2+1}$

Đặt $a=x^2;b=y^2;c=z^2\Rightarrow a+b+c=3$ Bất đẳng thức cần chứng minh trở thành

$\sum \frac{1}{\frac{2a^3}{a+1}-a+3}\leq 1\Leftrightarrow \sum \frac{a+1}{2a^3-a^2+2a+3}\leq 1\Leftrightarrow \frac{(a-1)^2(-2a^3+3a+3)}{2a^3-a^2+2a+3}\geq 0$ (1)

Không mất tính tổng quát giả sử: $a\geq b\geq c\Rightarrow a\geq 1\geq c$. Xét hai trường hợp:

TH1: $b+c\geq 1\Rightarrow a\leq 2$, lúc đó:

$-2a^3+3a+3>0;-2b^3+3b+3>0;-2c^3+3c+3>0$ nên (1) đúng

TH2: $b+c\leq 1\Rightarrow a\geq 2$, lúc đó:

$(2a^3-a^2+2a+3)-5(a+1)=2a^3-a^2-3a-2=a^3(2-\frac{1}{a}-\frac{3}{a^2}-\frac{2}{a^3})\geq a^3(2-\frac{1}{2}-\frac{3}{2^2}-\frac{2}{2^3})=\frac{a^3}{2}>0$

$\Rightarrow \frac{a+1}{2a^3-a^2+2a+3}\leq \frac{1}{5}$. Cần chứng minh:

$\frac{b+1}{2b^3-b^2+2b+3}+\frac{c+1}{2c^3-c^2+2c+3}\leq \frac{4}{5}$

Ta có bổ đề: Với mọi $0<x<1$, ta có: $\frac{x+1}{2x^3-x^2+2x+3}\leq \frac{2}{5}\Leftrightarrow 4x^3\geq (x+1)(2x-1)$

+TH1: Nếu $x\leq \frac{1}{2}$, ta có điều phải chứng minh

+TH2: Nếu $x\geq \frac{1}{2}$ ta có:

$4x^3-(x+1)(2x-1)\geq 4x^3-2(2x-1)=2(2x^3-2x+1)\geq 2(x^2-2x+1)=2(x-1)^2\geq 0$ (đúng)

Ta có điều cần chứng minh

Đạt tại: $a=b=c=1$